Difference between revisions of "2002 AMC 10A Problems/Problem 20"

(Solution)
(Solution 2)
(10 intermediate revisions by 4 users not shown)
Line 5: Line 5:
  
 
==Solution 1==
 
==Solution 1==
Since <math>AG</math> and <math>CH</math> are parallel, triangles <math>GAD</math> and <math>HCD</math> are similar. Hence, <math>CH/AG = CD/AD = 1/3</math>.
+
First we can draw an image.
 +
<asy>
 +
unitsize(0.8 cm);
  
Since <math>AG</math> and <math>JE</math> are parallel, triangles <math>GAF</math> and <math>JEF</math> are similar. Hence, <math>EJ/AG = EF/AF = 1/5</math>. Therefore, <math>CH/EJ = (CH/AG)/(EJ/AG) = (1/3)/(1/5) = \boxed{5/3}</math>. The answer is (D).
+
pair A, B, C, D, E, F, G, H, J;
 +
 
 +
A = (0,0);
 +
B = (1,0);
 +
C = (2,0);
 +
D = (3,0);
 +
E = (4,0);
 +
F = (5,0);
 +
G = (-1.5,4);
 +
H = extension(D, G, C, C + G - A);
 +
J = extension(F, G, E, E + G - A);
 +
 
 +
draw(A--F--G--cycle);
 +
draw(B--G);
 +
draw(C--G);
 +
draw(D--G);
 +
draw(E--G);
 +
draw(C--H);
 +
draw(E--J);
 +
 
 +
label("$A$", A, SW);
 +
label("$B$", B, S);
 +
label("$C$", C, S);
 +
label("$D$", D, S);
 +
label("$E$", E, S);
 +
label("$F$", F, SE);
 +
label("$G$", G, NW);
 +
label("$H$", H, W);
 +
label("$J$", J, NE);
 +
</asy>
 +
 
 +
Since <math>\overline{AG}</math> and <math>\overline{CH}</math> are parallel, triangles <math>\triangle GAD</math> and <math>\triangle HCD</math> are similar. Hence, <math>\frac{CH}{AG} = \frac{CD}{AD} = \frac{1}{3}</math>.
 +
 
 +
Since <math>\overline{AG}</math> and <math>\overline{JE}</math> are parallel, triangles <math>\triangle GAF</math> and <math>\triangle JEF</math> are similar. Hence, <math>\frac{EJ}{AG} = \frac{EF}{AF} = \frac{1}{5}</math>. Therefore, <math>\frac{CH}{EJ} = \left(\frac{CH}{AG}\right)\div\left(\frac{EJ}{AG}\right) = \left(\frac{1}{3}\right)\div\left(\frac{1}{5}\right) = \boxed{\frac{5}{3}}</math>. The answer is <math>\boxed{(D) 5/3}</math>.
  
 
==Solution 2==
 
==Solution 2==
As <math>\overline{JE}</math> is parallel to <math>\overline{AG}</math>, angles FJE and FGA are congruent. Also, angle F is clearly congruent to itself. From AA similarity, <math>\triangle AGF \sim \triangle EJF</math>; hence <math>\frac {AG}{JE} =5</math>. Similarly, <math>\frac {AG}{HC} = 3</math>. Thus, <math>\frac {HC}{JE} = \boxed{\frac {5}{3}\Rightarrow \text{(D)}}</math>.
+
As angle F is clearly congruent to itself, we get from AA similarity, <math>\triangle AGF \sim \triangle EJF</math>; hence <math>\frac {AG}{JE} =5</math>. Similarly, <math>\frac {AG}{HC} = 3</math>. Thus, <math>\frac {HC}{JE}=\left(\frac{AG}{JE}\right)\left(\frac{HC}{AG}\right) = \boxed{\frac {5}{3}\Rightarrow \text{(D)}}</math>.
 +
 
 +
==Solution 3==
 +
Assume an arbitrary value of <math>AG=15</math> WLOG. <math>\overline{AG}</math> and <math>\overline{CH}</math> are parallel, so <math>\triangle GAD</math> and <math>\triangle HCD</math> are similar. So, <math>\frac{AD}{CD}=3=\frac{GA}{HC}</math> which means <math>HC=5</math>. By the same logic, <math>JE=3</math>, so <math>\frac{HC}{JE}=\frac{5}{3}</math>.
  
 
==See Also==
 
==See Also==

Revision as of 22:04, 28 August 2021

Problem

Points $A,B,C,D,E$ and $F$ lie, in that order, on $\overline{AF}$, dividing it into five segments, each of length 1. Point $G$ is not on line $AF$. Point $H$ lies on $\overline{GD}$, and point $J$ lies on $\overline{GF}$. The line segments $\overline{HC}, \overline{JE},$ and $\overline{AG}$ are parallel. Find $HC/JE$.

$\text{(A)}\ 5/4 \qquad \text{(B)}\ 4/3 \qquad \text{(C)}\ 3/2 \qquad \text{(D)}\ 5/3 \qquad \text{(E)}\ 2$

Solution 1

First we can draw an image. [asy] unitsize(0.8 cm);  pair A, B, C, D, E, F, G, H, J;  A = (0,0); B = (1,0); C = (2,0); D = (3,0); E = (4,0); F = (5,0); G = (-1.5,4); H = extension(D, G, C, C + G - A); J = extension(F, G, E, E + G - A);  draw(A--F--G--cycle); draw(B--G); draw(C--G); draw(D--G); draw(E--G); draw(C--H); draw(E--J);  label("$A$", A, SW); label("$B$", B, S); label("$C$", C, S); label("$D$", D, S); label("$E$", E, S); label("$F$", F, SE); label("$G$", G, NW); label("$H$", H, W); label("$J$", J, NE); [/asy]

Since $\overline{AG}$ and $\overline{CH}$ are parallel, triangles $\triangle GAD$ and $\triangle HCD$ are similar. Hence, $\frac{CH}{AG} = \frac{CD}{AD} = \frac{1}{3}$.

Since $\overline{AG}$ and $\overline{JE}$ are parallel, triangles $\triangle GAF$ and $\triangle JEF$ are similar. Hence, $\frac{EJ}{AG} = \frac{EF}{AF} = \frac{1}{5}$. Therefore, $\frac{CH}{EJ} = \left(\frac{CH}{AG}\right)\div\left(\frac{EJ}{AG}\right) = \left(\frac{1}{3}\right)\div\left(\frac{1}{5}\right) = \boxed{\frac{5}{3}}$. The answer is $\boxed{(D) 5/3}$.

Solution 2

As angle F is clearly congruent to itself, we get from AA similarity, $\triangle AGF \sim \triangle EJF$; hence $\frac {AG}{JE} =5$. Similarly, $\frac {AG}{HC} = 3$. Thus, $\frac {HC}{JE}=\left(\frac{AG}{JE}\right)\left(\frac{HC}{AG}\right) = \boxed{\frac {5}{3}\Rightarrow \text{(D)}}$.

Solution 3

Assume an arbitrary value of $AG=15$ WLOG. $\overline{AG}$ and $\overline{CH}$ are parallel, so $\triangle GAD$ and $\triangle HCD$ are similar. So, $\frac{AD}{CD}=3=\frac{GA}{HC}$ which means $HC=5$. By the same logic, $JE=3$, so $\frac{HC}{JE}=\frac{5}{3}$.

See Also

2002 AMC 10A (ProblemsAnswer KeyResources)
Preceded by
Problem 19
Followed by
Problem 21
1 2 3 4 5 6 7 8 9 10 11 12 13 14 15 16 17 18 19 20 21 22 23 24 25
All AMC 10 Problems and Solutions

The problems on this page are copyrighted by the Mathematical Association of America's American Mathematics Competitions. AMC logo.png